3
$\begingroup$

I have this expression $(x \geq 1) \vee(x=0 \,\wedge\, y -z \geq 1)$ which I am solving over the nonnegative integers $x,y,z \in \mathbb{Z}_0^+$.

I suspect it is impossible to find a system of linear equalities which has the same solution set as my expression, but I'm having trouble proving this. Is it true? How do you prove this?

I know that the solution set to a system of linear inequalities must be convex --- that is, the segment between any two points in the solution set must consist entirely of solutions. I also know that the union of convex spaces is not, in general, convex. However, because we're dealing with integers, it seems at least sometimes possible to create a convex space that contains the integer points in the nonconvex union of convex spaces.

What I've tried so far is drawing a picture of the lattice of solutions. The entire ~octant beyond $x\geq 1$ is filled in, as is an upward-sloping half-plane $y-z \geq 1$ in $x=0$.

Intuitively, when I try to picture putting a supporting hyperplane straddling the $y-z \geq 1$ part and the $x \geq 1$ part, the slope doesn't work --- the distance between (0,a+1,a) and (1,0,a) keeps growing. But I don't know how to formalize this.


The broader context is I'm considering nonnegative integer solutions to (sort-of) linear inequalities $\sum_i \alpha_i x_i \leq -1$, where the coefficients are either $1,0,-1$ or $-\infty$. And trying to figure out in what cases there exists a system of linear inequalities whose solutions over the nonnegative integers are equivalent.

As a concrete case, in this question I'm considering $-\infty x - y + z \leq -1$, which is equivalent to the expression at the top of this page.

I define the solution to an equation involving negative infinites recursively by saying that the solution to $-\infty \cdot x_i + L(\vec{x}) \leq -1$ is the union of the solution to $x_i \geq 1$ and the solution to $L(\vec{x})\leq -1$; in the base case, there are no infinities.

$\endgroup$
5
  • $\begingroup$ Do you only allow inequalities with $\leq$, or also with $<$? And do you allow only finitely many inequalities, or also infinitely many? $\endgroup$ Commented May 24, 2024 at 21:22
  • $\begingroup$ @Servaes Only $\leq$, and in fact specifically only the inequality $\leq -1$. And only finitely many inequalities. $\endgroup$ Commented May 24, 2024 at 21:36
  • $\begingroup$ Oh, I see what you mean actually. The resulting equivalent system may have any sort of inequality (though since we're dealing with integer solutions, it's possible we don't lose anything by restricting to $\leq$, not sure.) $\endgroup$ Commented May 24, 2024 at 21:38
  • $\begingroup$ Are you able to impose finite bounds on the variables? $\endgroup$ Commented May 25, 2024 at 0:08
  • $\begingroup$ Unfortunately, I don't have a bound on the variables. $\endgroup$ Commented May 26, 2024 at 22:13

1 Answer 1

3
$\begingroup$

The solution set of a finite system of linear inequalities of the form $$ax+by+cz\geq d,$$ is a convex and closed set. Your set contains $(0,1,0)$ and $(1,1,z)$ for every integer $z>0$, and so its convex hull contains $(\tfrac1z,1,1)$ for every integer $z>0$. Then the closure of its convex hull also contains $(0,1,1)$. This shows that your set is not the solution set of a finite system of linear inequalities.

$\endgroup$
3
  • $\begingroup$ To ask about a detail --- I guess because convex hulls and closures are minimal examples of their type, it follows that every closed convex set containing the integer solutions to my equation also contains (0,1,1), therefore every linear system that includes my integer solutions also includes (0,1,1). Is that right? $\endgroup$ Commented May 24, 2024 at 22:03
  • $\begingroup$ @user326210 That is exactly right. $\endgroup$ Commented May 26, 2024 at 15:48
  • $\begingroup$ Ah, okay, I now better understand your comment on $\leq$ vs $<$. If the system involves strict inequalities, it is possible the solution set is not closed. Is there any way to modify the argument here to say that there is no set of linear inequalities, even allowing strict inequality, whose integer solutions are the same as the integer solutions to $-\infty x -y+z\leq -1$ ($x,y,z,\geq 0$)? $\endgroup$ Commented May 26, 2024 at 22:17

You must log in to answer this question.

Start asking to get answers

Find the answer to your question by asking.

Ask question

Explore related questions

See similar questions with these tags.